K
Khách

Hãy nhập câu hỏi của bạn vào đây, nếu là tài khoản VIP, bạn sẽ được ưu tiên trả lời.

\(\frac{a}{1+b^2}=a-\frac{ab^2}{1+b^2}\ge a-\frac{ab^2}{2b}=a-\frac{ab}{2}\)

Tương tự các vế còn lại ta có

\(VT\ge a+b+c-\frac{ab+bc+ca}{2}\ge3-\frac{3}{2}=\frac{3}{2}\)

Dấu "=" xảy ra khi a=b=c=1

NV
6 tháng 5 2021

Ta chứng minh BĐT sau với các số dương:

\(\dfrac{1}{x}+\dfrac{1}{y}\ge\dfrac{4}{x+y}\)

Thật vậy, BĐT tương đương: \(\dfrac{x+y}{xy}\ge\dfrac{4}{x+y}\Leftrightarrow\left(x+y\right)^2\ge4xy\)

\(\Leftrightarrow x^2-2xy+y^2\ge0\Leftrightarrow\left(x-y\right)^2\ge0\) (luôn đúng)

Áp dụng:

\(\dfrac{1}{a}+\dfrac{1}{b}\ge\dfrac{4}{a+b}\) ; \(\dfrac{1}{b}+\dfrac{1}{c}\ge\dfrac{4}{b+c}\) ; \(\dfrac{1}{c}+\dfrac{1}{a}\ge\dfrac{4}{c+a}\)

Cộng vế với vế:

\(2\left(\dfrac{1}{a}+\dfrac{1}{b}+\dfrac{1}{c}\right)\ge\dfrac{4}{a+b}+\dfrac{4}{b+c}+\dfrac{4}{c+a}\)

\(\Leftrightarrow\dfrac{1}{a}+\dfrac{1}{b}+\dfrac{1}{c}\ge\dfrac{2}{a+b}+\dfrac{2}{b+c}+\dfrac{2}{c+a}\)

NV
6 tháng 5 2021

b.

Ta có:

\(\dfrac{1}{a}+\dfrac{1}{b}\ge\dfrac{4}{a+b}\Rightarrow\dfrac{3}{a}+\dfrac{3}{b}\ge\dfrac{12}{a+b}\) (1)

\(\dfrac{1}{b}+\dfrac{1}{c}\ge\dfrac{4}{b+c}\Rightarrow\dfrac{2}{b}+\dfrac{2}{c}\ge\dfrac{8}{b+c}\) (2)

\(\dfrac{1}{c}+\dfrac{1}{a}\ge\dfrac{4}{c+a}\) (3)

Cộng vế với vế (1); (2) và (3):

\(\dfrac{4}{a}+\dfrac{5}{b}+\dfrac{3}{c}\ge4\left(\dfrac{3}{a+b}+\dfrac{2}{b+c}+\dfrac{1}{c+a}\right)\) (đpcm)

Dấu "=" xảy ra khi \(a=b=c\)

9 tháng 8 2019

2) Theo nguyên lí Dirichlet, trong ba số \(a^2-1;b^2-1;c^2-1\) có ít nhất hai số nằm cùng phía với 1.

Giả sử đó là a2 - 1 và b2 - 1. Khi đó \(\left(a^2-1\right)\left(b^2-1\right)\ge0\Leftrightarrow a^2b^2-a^2-b^2+1\ge0\)

\(\Rightarrow a^2b^2+3a^2+3b^2+9\ge4a^2+4b^2+8\)

\(\Rightarrow\left(a^2+3\right)\left(b^2+3\right)\ge4\left(a^2+b^2+2\right)\)

\(\Rightarrow\left(a^2+3\right)\left(b^2+3\right)\left(c^2+3\right)\ge4\left(a^2+b^2+1+1\right)\left(1+1+c^2+1\right)\) (2)

Mà \(4\left[\left(a^2+b^2+1+1\right)\left(1+1+c^2+1\right)\right]\ge4\left(a+b+c+1\right)^2\) (3)(Áp dụng Bunhicopxki và cái ngoặc vuông)

Từ (2) và (3) ta có đpcm.

Sai thì chịu

9 tháng 8 2019

Xí quên bài 2 b:v

b) Không mất tính tổng quát, giả sử \(\left(a^2-\frac{1}{4}\right)\left(b^2-\frac{1}{4}\right)\ge0\)

Suy ra \(a^2b^2-\frac{1}{4}a^2-\frac{1}{4}b^2+\frac{1}{16}\ge0\)

\(\Rightarrow a^2b^2+a^2+b^2+1\ge\frac{5}{4}a^2+\frac{5}{4}b^2+\frac{15}{16}\)

Hay \(\left(a^2+1\right)\left(b^2+1\right)\ge\frac{5}{4}\left(a^2+b^2+\frac{3}{4}\right)\)

Suy ra \(\left(a^2+1\right)\left(b^2+1\right)\left(c^2+1\right)\ge\frac{5}{4}\left(a^2+b^2+\frac{1}{4}+\frac{1}{2}\right)\left(\frac{1}{4}+\frac{1}{4}+c^2+\frac{1}{2}\right)\)

\(\ge\frac{5}{4}\left(\frac{1}{2}a+\frac{1}{2}b+\frac{1}{2}c+\frac{1}{2}\right)^2=\frac{5}{16}\left(a+b+c+1\right)^2\) (Bunhiacopxki) (đpcm)

Đẳng thức xảy ra khi \(a=b=c=\frac{1}{2}\)

22 tháng 3 2019

Áp dụng bất đẳng thức Cô-si ta có:

\(\dfrac{a^2}{b^3}+\dfrac{1}{a}+\dfrac{1}{a}\ge\sqrt[3]{\dfrac{a^2}{b^3}.\dfrac{1}{a}.\dfrac{1}{a}}=\dfrac{3}{b}\)

\(\dfrac{c^2}{a^3}+\dfrac{1}{c}+\dfrac{1}{c}\ge\sqrt[3]{\dfrac{c^2}{a^3}.\dfrac{1}{c}.\dfrac{1}{c}}=\dfrac{3}{a}\)

\(\dfrac{c^2}{a^3}+\dfrac{1}{c}+\dfrac{1}{c}\ge\sqrt[3]{\dfrac{c^2}{a^3}.\dfrac{1}{c}.\dfrac{1}{c}}=\dfrac{3}{a}\)

Cộng theo vế ta được:

\(\dfrac{a^2}{b^3}+\dfrac{b^2}{c^3}+\dfrac{a^2}{a^3}+\dfrac{2}{a}+\dfrac{2}{b}+\dfrac{2}{c}\ge3\left(\dfrac{1}{a}+\dfrac{1}{b}+\dfrac{1}{c}\right)\)

\(\Leftrightarrow\dfrac{a^2}{b^3}+\dfrac{b^2}{c^3}+\dfrac{c^2}{a^3}\ge\dfrac{1}{a}+\dfrac{1}{b}+\dfrac{1}{c}\)

8 tháng 5 2018

\(\frac{a^2}{b^3}+\frac{b^2}{c^3}+\frac{c^2}{a^3}=\frac{1}{b}+\frac{1}{c}+\frac{1}{a}\)

=> \(\frac{a^2}{b^3}+\frac{b^2}{c^3}+\frac{c^2}{a^3}=\frac{1}{a}+\frac{1}{b}+\frac{1}{c}\)

20 tháng 5 2018

\(\frac{a^2}{b^3}+\frac{1}{a}+\frac{1}{a}\ge3\cdot\frac{1}{b}\)

30 tháng 7 2018

\(\frac{1}{a^2}+\frac{1}{b^2}=\frac{a}{a^3}+\frac{1}{b^2}\ge\frac{\left(\sqrt{a}+1\right)^2}{a^3+b^2}\ge\frac{4\sqrt{a}}{a^3+b^2}\)

Cứ tiếp tục như vậy ta sẽ có đpcm. dấu = xảy ra khi a=b=c=1

21 tháng 4 2020

ta có

\(\frac{a}{1+b^2}=a-\frac{ab^2}{1+b^2}\ge a-\frac{ab^2}{2b}=a-\frac{ab}{2}\left(AM-GM\right)\)

tương tự ta có

\(\frac{a}{1+b^2}+\frac{b}{1+c^2}+\frac{c}{1+a^2}\ge\left(a+b\ge+c\right)-\frac{1}{2}\left(ab+bc+ca\right)\ge\frac{3}{2}\)

do \(ab+bc+ca\le\frac{\left(a+b+c\right)^2}{3}=3\)

31 tháng 8 2018

Ta có: Theo bất đẳng thức cauchy schwarz và bất đẳng thức cauchy với a;b;c>0 ta có:

\(\dfrac{1}{a^2}+\dfrac{1}{a^2}=\dfrac{\left(\sqrt{a}\right)^2}{a^3}+\dfrac{1}{a^2}\ge\dfrac{\left(\sqrt{a}+1\right)^2}{a^3+a^2}\ge\dfrac{4\sqrt{a}}{a^3+a^2}\)(1)

Tương tự \(\dfrac{1}{b^2}+\dfrac{1}{b^2}\ge\dfrac{4\sqrt{b}}{b^3+b^2}\left(2\right);\dfrac{1}{c^2}+\dfrac{1}{c^2}\ge\dfrac{4\sqrt{c}}{c^3+c^2}\left(3\right)\)

Cộng từng vế (1) ;(2);(3) vế theo vế rồi chia hai vế cho 2 ta có đpcm

1 tháng 9 2018

Sai rồi nhé

KT lại đi bạn ơi

9 tháng 9 2018

ta có : \(\dfrac{1}{a^2}+\dfrac{1}{b^2}=\dfrac{a}{a^3}+\dfrac{1}{b^2}\ge\dfrac{\left(\sqrt{a}+1\right)^2}{a^3+b^2}=\dfrac{a^2+2\sqrt{a}+1}{a^3+b^2}\ge\dfrac{4\sqrt{a}}{a^3+b^2}\)

làm tương tự ta có : \(\dfrac{1}{b^2}+\dfrac{1}{c^2}\ge\dfrac{4\sqrt{b}}{b^3+c^2}\)\(\dfrac{1}{c^2}+\dfrac{1}{a^2}\ge\dfrac{4\sqrt{c}}{c^3+a^2}\)

cộng quế theo quế \(\Rightarrow\) (đpcm)

9 tháng 9 2018

bạn làm sai rồi

cái dòng đầu tiên í

\(\dfrac{\left(\sqrt{a}+1\right)^2}{a^3+b^2}\ne\dfrac{a^2+2\sqrt{a}+1}{a^3+b^2}\)